scottflanary
Thanks Received: 0
Vinny Gambini
Vinny Gambini
 
Posts: 7
Joined: June 04th, 2011
 
 
 

Q17 - Politician: Some of my opponents

by scottflanary Sun Jul 17, 2011 2:52 pm

Hello! For some reason, I cannot wrap my head around this question. Correct answer (A) was one of the first answer choices I marked off because I read "excessive public expenditure on social programs" as the opponents' arguments (when taken in context that the Politician thinks INSTEAD of that, they should talk about the main cause of deficit spending). Can someone please help?

I had my answer choices originally narrowed down to (D) and (E), selecting (E) after many minutes of frustration (aka total guess).
User avatar
 
LSAT-Chang
Thanks Received: 38
Atticus Finch
Atticus Finch
 
Posts: 479
Joined: June 03rd, 2011
 
 
trophy
Most Thankful
trophy
First Responder
 

Re: Q17 - Politician: Some of my opponents

by LSAT-Chang Sun Jul 17, 2011 6:55 pm

I also need help with this one..

I narrowed it down to (A) and (D), and ended up picking (D) because of the following reasons:

1) the argument starts off by saying that the opponents argued in favor of "reducing social spending". The next sentence (author speaking) kind of "rephrases" what the opponents said and does make it more extreme by quoting "instead of arguing that there is EXCESSIVE public expenditure..." well, this popped right at me since if I'm arguing that there is a need to "reduce" my shopping budget, that doesn't mean that I'm "excessively" spending on shopping. I could still be spending just a little money, but just that I need to "reduce" it more from however much I am currently spending (whether that be low or high $$ amount). This totally matched up with the answer choice (D) in that the opponents' views are being portrayed as more extreme than they really are, since they never explicitly said that there is an EXCESSIVE public expenditure on social programs!

2) the only reason I didn't cross out (A) was because the author's conclusion is: that it is unwarranted to reduce social expenditure. But I couldn't really find a direct "support" for this conclusion. I couldn't answer "why does the author believe that it is unwarranted to reduce social expenditure?" Because the government is bloated with bureaucrats and self-aggrandizing politicians?? It didn't really make sense to me. Then I thought, "oh is this what (A) is getting at? Is the author not addressing the arguments advanced by the politician's opponents?" But he does.. the first sentence does explicitly say the opponent's argument. Then it got really confusing. So I just went with (D) without thinking further about it since (A) felt like one of those trap answer choices that makes you think so hard and come up with all these assumptions that will hold for (A) to be true, but in the end, you've wasted so much time thinking about making the answer choice "possible" when in fact, it isn't possible. So I went with (D).

I still don't get why (A) would be correct and am dying to know why (D) is not the flaw in the argument... (couldn't wait till my next office hour on Tuesday). Please help meeeee! :shock:
 
rzaman
Thanks Received: 4
Vinny Gambini
Vinny Gambini
 
Posts: 10
Joined: October 23rd, 2010
 
This post thanked 3 times.
 
 

Re: Q17 - Politician: Some of my opponents...

by rzaman Tue Jul 19, 2011 2:43 pm

The way I interpreted this argument is that the politician does not actually address his opponent's arguments. He basically tells them that "Instead of arguing whatever you are arguing, you should be focused on what I think is the issue -- self-aggrandizing politicians. And therefore, I'm going to tell you that whatever you propose is unwarranted."

And this issue here is that how can the politician say that the opponent's argument is unwarranted if he hasn't actually engaged with that argument? Just stating his opponent's arguments does not mean he is actually addressing them.

Changsoyeon, I believe what you are pointing to is a term shift between arguing something should be reduced and stating that the spending is currently excessive. I think the reason why it works in the argument and why you can argue that there isn't a term shift here is because if something needs to be reduced, that includes assuming that whatever amount is currently being spent is excessive to what should be the amount.

In terms of your counterexample that needing to reduce your shopping budget does not mean you are spending excessively spending on shopping -- I get that but I think you mean you are not excessively spending on shopping relative to something else. Taken by itself, if you need to reduce your shopping budget, you are currently spending an excess to what you need to spend.
User avatar
 
LSAT-Chang
Thanks Received: 38
Atticus Finch
Atticus Finch
 
Posts: 479
Joined: June 03rd, 2011
 
 
trophy
Most Thankful
trophy
First Responder
 

Re: Q17 - Politician: Some of my opponents...

by LSAT-Chang Tue Jul 19, 2011 3:27 pm

This is an awesome response. Thank you so much! It cleared up everything for me. I understood the term "excessive" to mean just WAY too much, but you are right, the literal meaning does mean that I am just spending (1 cent or 100 dollars) an amount over my limit and so it clearly makes sense that I would have to "reduce" the amount. Thanks so much!!!
User avatar
 
bbirdwell
Thanks Received: 864
Atticus Finch
Atticus Finch
 
Posts: 803
Joined: April 16th, 2009
 
 
 

Re: Q17 - Politician: Some of my opponents...

by bbirdwell Tue Jul 19, 2011 5:15 pm

Great discussion guys! Thanks for chiming in, rzaman.
I host free online workshop/Q&A sessions called Zen and the Art of LSAT. You can find upcoming dates here: http://www.manhattanlsat.com/zen-and-the-art.cfm
 
jamiejames
Thanks Received: 3
Atticus Finch
Atticus Finch
 
Posts: 116
Joined: September 17th, 2011
 
 
 

Re: Q17 - Politician: Some of my opponents

by jamiejames Tue May 29, 2012 5:59 pm

(A) correct for reasons stated above

(b) wrong because that isn't what's happening, and don't think that by calling some politicians self-aggrandizing and bloated, that he is calling his opponents this, despite that they may very well be in the government.

(c) the stim says that the MAIN cause is x, which implies that there are other causes which are not as important as this one

(d) simply doesn't do this anywhere in the stim

(e) while true, making clear what excessive spending is wouldn't help, and not defining it doesn't hurt the argument.
 
KakaJaja
Thanks Received: 1
Forum Guests
 
Posts: 37
Joined: May 17th, 2012
 
 
 

Re: Q17 - Politician: Some of my opponents

by KakaJaja Tue Sep 11, 2012 8:59 pm

According to the politician, his opponent's opinion is:
"The spending on social program is too much --> We should reduce social spending"
In response, the politician said "The main cause of deficit is self-aggrandizing --> we should not reduce social spending"

So I think A) is correct because the opponent wants to reduce social spending simply because it is too much, the basis of his argument is not "social spending causes deficit". So the politician is actually attacking sth he himself created, not the problem put forward by his opponent.

Is my reasoning correct?

If it is correct, I have a question, does "theoretical grounds" in the second line imply anything? THANKS!
 
liq419
Thanks Received: 0
Vinny Gambini
Vinny Gambini
 
Posts: 4
Joined: June 02nd, 2018
 
 
 

Re: Q17 - Politician: Some of my opponents

by liq419 Fri Sep 20, 2019 5:11 pm

Here's my take :D (breaking arg down in abstract terms)

STIM:

My opponents believe:
(Conclusion) we should reduce social expenditure
(Premise) because of A*
VS.
I believe:
(conclusion) we should NOT reduce social expenditure
(Premise) because my opponent should focus on B* instead of A


-------
Flaw:
Just because they (opponents) should focus on B instead of A, does that make them wrong?
No. Author should have attacked A (the reasoning of the opponent) instead of diverting the attention to B.
------

Choices:
(A) correct. Author didn't even address A.
(B) wrong. Author didn't mention the characters of his opponents. It's not Ad Hominem.
(c) wrong. Author said "main cause", not the "only" cause - leaving room for interpretation (of other causes).
(d) wrong. No extreme language was used.
(e) wrong. Author isn't required to define a term.


------
A* = there's excessive spending
B* = "main cause of deficit spending...politicians." (the whole chunk- that's pretty useless)